Search found 12 matches


My weaknesses were in probabilities and Venn Diagrams. So I took two days each to review them. I am good with integers, geometry and algebra, so I reviewed them by taking practice tests. As I said, I always had scores in quant above 47. Even in quant with my 47 score in the first test, I didn't even...

by Cdawg

Thu Apr 22, 2010 9:20 am
Forum: I just Beat The GMAT!
Topic: beaten again 620->640
Replies: 6
Views: 2317

beaten again 620->640

LoL I got beaten...AGAIN! 640 (Q42, V36) again I am devastated. My previous score was 620 (Q47, V29). I could have saved weeks of prep for such a shitty result. score change: quant: -5 verbal +7 I have no idea how I could score so low in Quant, which is actually my strength. It's so frustrating. I f...

by Cdawg

Tue Apr 20, 2010 9:19 am
Forum: I just Beat The GMAT!
Topic: beaten again 620->640
Replies: 6
Views: 2317

Is X negative? 1. x^3(1-x^2)<0 2. x^2-1<0 Please explain your answer.. Ron, how do I go about applying your strategy of picking numbers combining both statements to check for option C or E right in the begining? Am looking for a 2 min solution here. Look forward to your reply. C There are two parts...

by Cdawg

Tue Apr 13, 2010 9:57 am
Forum: Data Sufficiency
Topic: Number Properties -
Replies: 4
Views: 1255

eaakbari wrote:
sanju09 wrote: (1) Chord PQ is same as radius and area of the circle it belongs to.
.

I cannot comprehend this statement. Someone please explain
chord = radius = area

by Cdawg

Tue Apr 13, 2010 9:12 am
Forum: Data Sufficiency
Topic: the circle it belongs to
Replies: 7
Views: 1436

How far is point P from point Q? (1) Chord PQ is same as radius and area of the circle it belongs to. (2) Major-arc PQ of the circle it belongs to is 5/3. Statement 1: if Area = radius = chord, it follows that A = r = Pi*r^2 => r=1/Pi so chord is 1/Pi No other circle fulfills the requirement that a...

by Cdawg

Tue Apr 13, 2010 9:12 am
Forum: Data Sufficiency
Topic: the circle it belongs to
Replies: 7
Views: 1436

I got D. Can someone verify my math. Is y<(x+z)/2. = Is 2y<x + z ? 1. y - x < z - y = Add y to both sides --> 2y - x < z = Add x to both sides -- > 2y < x + z 1 is sufficient 2. z - y > (z - x)/2 = Multiply both sides by 2 --> 2(z - y) > z - x --> 2z - 2y > z - x = Add both sides by -2z --> -2y > -...

by Cdawg

Mon Apr 12, 2010 9:42 am
Forum: Data Sufficiency
Topic: Is y < x+z/2
Replies: 9
Views: 1939

In order for |N+5|=5 to be true N must either be 0 or -10 (1) N^2 not equal 0, tells you that N is not 0 (since 0^2 equals 0), therefore N must be -10 => sufficient (2) N^2 + 10N = 0 tells you that N can be either -10 or 0, which doesn't help you much in determining N, => insufficient so the answer ...

by Cdawg

Mon Apr 12, 2010 9:27 am
Forum: Data Sufficiency
Topic: |N+5|=5, what is the value of N?
Replies: 5
Views: 1554

Inequalities: (1) x-y > 0 (2) y-x < 0 =!= 0 < x-y OR x-y > 0 as sanju09 said, they are the same test: x=1 y=-2 => 1-(-2)=3, thus statements hold, and x+y=-1 (neg) but statements also hold when x=2 y=1, and X+y=3 (pos) thus you can't say whether they are + or - ANSWER: E

by Cdawg

Mon Apr 12, 2010 9:21 am
Forum: Data Sufficiency
Topic: +?
Replies: 6
Views: 1436

(1) maximum she could have walked is 12.8km, thus it took her certainly longer => sufficient

(2) in 2h=120min the furthest she could have walked is 120/9= 13.33km => sufficient

both statements allow to conlude that it took her longer than 2h to walk 16km

Answer: D

by Cdawg

Mon Apr 12, 2010 9:09 am
Forum: Data Sufficiency
Topic: time \ dist
Replies: 6
Views: 1372

(1) x could be 2, which is prime, but also 4, which isn't prime => insufficient (2) x could be 1;4;8;10;etc. which aren't prime, but also be 2, which is prime = > insufficient together: if x is even AND not divisible by any other odd number than 1, x could be 2, which is prime, but also be 4, which ...

by Cdawg

Mon Apr 12, 2010 8:47 am
Forum: Data Sufficiency
Topic: x a prime
Replies: 10
Views: 1656

(1) obviously the statement basically states that k is an integer that is divisible by 3 (since 2k/3 is even) and that integer k cannot be smaller than 2, because if k=2, the statement wouldn't hold). 2*2/3 = not evenly divisible Thus, k itself must be a integer that is divisible by 3, but larger th...

by Cdawg

Mon Apr 12, 2010 8:21 am
Forum: Data Sufficiency
Topic: k evenly
Replies: 27
Views: 2919

I just got owned by the Gmat!!

620! quant 47 verbal 29 LOL - F*** my Life!!!!! (excuse me) I don't know what happend! I have never ever scored below 30, not even during the first practice test I did without even looking at the books once. Sentence corrections and time killed me. Quant went so so, but in quant I am just on top of ...

by Cdawg

Thu Mar 11, 2010 9:53 am
Forum: I just Beat The GMAT!
Topic: I just got owned by the Gmat!!
Replies: 2
Views: 1661